logo

에르미트 다항식의 직교성 📂함수

에르미트 다항식의 직교성

정리

에르미트 다항식 $\left\{ H_{n} \right\}_{n=0}^{\infty}$은 구간 $(-\infty, \infty)$에서 가중 함수 $w(x)=e^{-x^{2}}$에 대해서 직교한다.

$$ \braket{ H_{n} | H_{m} }_{e^{-x^{2}}} =\int_{-\infty}^{\infty}e^{-x^{2}}H_{n}(x)H_{m}(x)dx=\sqrt{\pi}2^{n}n!\delta_{nm} $$

이때 $\delta_{nm}$은 크로네커 델타이다.

증명

경우 1: $n=m$

미분 연산자를 $D = \dfrac{d}{dx}$라고 표기하자.

$$ \int_{-\infty}^{\infty} e^{-x^{2}}H_{n}(x)H_{n}(x)dx $$

에르미트 다항식

$$ H_{n}(x) = (-1)^{n}e^{x^{2}}\frac{d^{n}}{dx^{n}}e^{-x^{2}} = (-1)^{n}e^{x^{2}}D^{n}e^{-x^{2}} $$

위 식에서 앞의 $H_{n}(x)$만 풀어서 써보면 다음과 같다.

$$ \begin{align*} \int_{-\infty}^{\infty} e^{-x^{2}}(-1)^{n}e^{x^{2}}\left[D^{n}e^{-x^{2}} \right]H_{n}(x)dx &= \int_{-\infty}^{\infty} (-1)^{n}\left[D^{n}e^{-x^{2}} \right]H_{n}(x)dx \end{align*} $$

위 식을 부분적으로 풀어보면 아래의 식을 얻는다.

$$ \begin{align*} &\int_{-\infty}^{\infty} (-1)^{n}\left[D^{n}e^{-x^{2}} \right]H_{n}(x)dx \\ &=\left[ (-1)^{n}\left(D^{n-1}e^{-x^{2}}\right)H_{n}(x) \right]_{-\infty}^{\infty}-\int_{-\infty}^{\infty}(-1)^{n}\left[ D^{n-1}e^{-x^{2}}\right]H^{\prime}_{n}(x)dx \tag{1} \end{align*} $$

이때 첫번째 항은 $\lim \limits_{x\rightarrow \pm\infty}D^{n-1}e^{-x^{2}}=0$이므로 $0$이다. 이 극한이 $0$으로 수렴하는 이유는 임의의 $n$에 대해서

$$ D^{n}e^{-x^{2}}=p(x)e^{-x^{2}} $$ 로 나타나기 때문이다. 여기에서 $p(x)$는 임의의 다항식이다. $x \rightarrow \pm \infty$일 때 다항식이 발산하는 속도보다 $e^{-x^{2}}$가 $0$으로 가는 속도가 훨씬 빠르므로 극한이 $0$으로 수렴한다.

에르미트 다항식의 재귀 관계 $$ H_{n}^{\prime}(x) =2nH_{n-1}(x) $$

또한 에르미트 다항식의 재귀관계에 의해서 $(1)$은 다음과 같다.

$$ -2n\int_{-\infty}^{\infty}(-1)^{n}\left[ D^{n-1}e^{-x^{2}} \right]H_{n-1}(x)dx $$

방금과 같은 논리로 부분적분을 한 번 더 하면 다음을 얻는다.

$$ (-1)^{2}2^{2}n(n-1)\int_{-\infty}^{\infty}(-1)^{n}\left[ D^{n-2}e^{-x^{2}} \right]H_{n-2}(x)dx $$

따라서 부분적분을 $n$번 하면 아래의 식을 얻는다.

$$ (-1)^{n}2^{n}n!\int_{-\infty}^{\infty}(-1)^{n}e^{-x^{2}}H_{0}(x)dx $$

$H_{0}(x)=1$이므로 아래와 같이 정리된다.

$$ 2^{n}n!\int_{-\infty}^{\infty}e^{-x^{2}}dx $$

위 적분은 가우스 적분으로 그 값은 $\sqrt{\pi}$이다. 그러므로 다음을 얻는다.

$$ \int_{-\infty}^{\infty} e^{-x^{2}}H_{n}(x)H_{n}(x)dx=\sqrt{\pi}2^{n}n! $$

경우 2: $n\ne m$

일반성을 잃지 않고 $n \gt m$이라고 하자.

$$ \int_{-\infty}^{\infty}e^{-x^{2}}H_{n}(x)H_{m}(x)dx $$

여기서 $H_{n}(x)$만 풀어서 적으면 다음과 같다.

$$ \int_{-\infty}^{\infty}e^{-x^{2}}(-1)^{n}e^{x^{2}}\left[D^{n}e^{-x^{2}}\right]H_{m}(x)dx=\int_{-\infty}^{\infty}(-1)^{n}\left[D^{n}e^{-x^{2}}\right]H_{m}(x)dx $$

앞의 $n=m$인 경우에 대한 증명에서와 같은 방법으로 $m$번 부분적분하면 아래의 식을 얻는다.

$$ (-1)^{n+m}2^{m}m!\int_{-\infty}^{\infty}\left[ D^{n-m}e^{-x^{2}} \right]\cdot 1 dx $$

여기서 한 번 더 부분적분하면 다음과 같다.

$$ \begin{align*} &(-1)^{n+m}2^{m}m!\int_{-\infty}^{\infty}\left[ D^{n-m}e^{-x^{2}} \right]\cdot 1 dx \\ &= (-1)^{n+m}2^{m}m!\left(\left[D^{n-m-1}e^{-x^{2}} \right]_{-\infty}^{\infty}+2(m+1)\int_{-\infty}^{\infty}\left[ D^{n-m-1}e^{-x^{2}}\right]\cdot 0 dx \right) \end{align*} $$

첫번째 항은 위의 증명에서 설명했듯이 $0$이고 두번째 항도 $0$이다. 그러므로 다음을 얻는다.

$$ \int_{-\infty}^{\infty}e^{-x^{2}}H_{n}(x)H_{m}(x)dx=0, \quad n \ne m $$